Magnetism seems absolute despite being relativistic effect of electrostatics

In summary: Then you switch to the electron/test charge frame and measure the magnetic field there. You would measure the magnetic field to be zero. This is because the magnetic field in the lab frame is due to the currents in the wire, and the currents have been turned off in the electron/test-charge frame.
  • #71
universal_101 said:
If this is how you see it, then how are you able to explain different scenario with Lorentz transformation. Or, can LT be applied on different scenarios/situations too ?

I'm sure you know this already, but then I can't seem to figure out why are you implying anything like this.
Sorry, I don't know if there is a language barrier, but I cannot really parse your post. I will answer what I guess is your question, but if I guess wrong please try to clarify your question carefully.

The LT can be applied to any scenario to generate an infinite number of other scenarios which are, in fact, physically identical to the original scenario. However, two arbitrary scenarios are not necessarily related to each other via a LT. In your case, (1) and (2) are not related by a LT.
 
Physics news on Phys.org
  • #72
DaleSpam said:
Here is probably the best resource for this question:
http://physics.weber.edu/schroeder/mrr/MRRtalk.html

Your scenarios are explicitly covered in the section "Magnetism as a Consequence of Length Contraction".

DaleSpam said:
Sorry, I don't know if there is a language barrier, but I cannot really parse your post. I will answer what I guess is your question, but if I guess wrong please try to clarify your question carefully.

The LT can be applied to any scenario to generate an infinite number of other scenarios which are, in fact, physically identical to the original scenario. However, two arbitrary scenarios are not necessarily related to each other via a LT. In your case, (1) and (2) are not related by a LT.

You described the two scenarios using LT, and now you are implying that the two scenarios are different. But to have a debate, you should stand with only one of the following.

Either the scenarios are different, or, they can be explained by LT.

And if you still think they are different, then please explain, why does the link you provided uses LT to explain different scenarios.
 
  • #73
universal_101 said:
You described the two scenarios using LT, and now you are implying that the two scenarios are different. But to have a debate, you should stand with only one of the following.
As I stated above, I mis-read your OP. The scenarios that I described using the LT correspond to your scenario (1) and to the LT of (1). They do not correspond to your scenario (2). I identified the modification that you would need to make to (2) in order to make it physically equivalent to (1).

universal_101 said:
Either the scenarios are different, or, they can be explained by LT.
They are different.

universal_101 said:
And if you still think they are different, then please explain, why does the link you provided uses LT to explain different scenarios.
It doesn't. The link I provided uses the LT to analyze the same scenario from two different reference frames. The scenario analyzed in the link is not the same as your (1) or (2).
 
  • #74
There was a discussion here about almost the same topic a long time ago:
https://www.physicsforums.com/archive/index.php/t-327854.html

I agree with what I read there on the first page; I haven't read the whole discussion.
Note: Also dalespam participated. Dalespam, do you agree with your comments of then?
 
Last edited:
  • #75
DrGreg said:
Attached to the bottom of this post is a diagram to help explain things. As was mentioned earlier in this thread, one way to approach the problem is to consider it a variant of the ladder paradox, and consider the different definitions of simultaneity.

But my approach here considers length contraction only. And I am going to consider a complete circuit: not just a single wire with a left-to-right electron flow, but also a return wire with a right-to-left flow. Apart from the ends of the wires, we keep the two wires far apart so they have negligible influence on each other. The diagram is a highly idealised simplification, considering just 16 electrons in the circuit. The ends of the wires should be in contact with each other but I've drawn them as separated to keep the diagram simple.

The top left part of the diagram shows the wires with no current flowing, in the rest-frame of the wires. 16 electrons equally spread out along the wire.

The top right part of the diagram again shows the wires with no current flowing, but now in a frame moving at the velocity that electrons would flow in the bottom wire if the current were on. We see length contraction as indicated by the yellow arrows. I'm assuming a Lorentz factor γ=2. So far so good.

The two bottom diagrams now show what happens when the current is flowing.

In the bottom left diagram, as we are told the wires remain electrically neutral, there must still be 16 electrons in the wires. There's no reason for the electrons to bunch together anywhere, they will remain spread out around the whole circuit as shown.

Finally, let's look at the bottom right diagram, which I think some people are having difficulty to imagine. We already know what happens to the red positive ions, their separation contracts just as before. The electrons in the lower wire are now stationary, so their separation must be larger than the bottom left diagram as shown. On the other hand, the electrons in the upper wire are moving faster than in bottom left diagram, so their separation must be less than in bottom left diagram. No electrons have escaped so the total number of electrons in circuit must still be 16. But now there are fewer electrons in the lower wire and more in the upper wire. So the lower wire has a positive charge and the upper wire has a negative charge.

DrGreg, stunning diagrams! But I disagree on the physics.

Some questions: why aren’t the electrons allowed to bunch together in the bottom left but they are allowed to bunch in the bottom right picture?

Referring to the bottom left picture, Biot-Savart tells me there’s a magnetic field present. Can you show me how length contraction is responsible for this magnetic field?
 
  • #76
Per Oni said:
Some questions: why aren’t the electrons allowed to bunch together in the bottom left but they are allowed to bunch in the bottom right picture?
The bottom left diagram is symmetrical: the upper wire is identical to the bottom wire apart from the direction of the electron flow. Therefore there's no reason for the electron distribution to be different in the two wires. The electrons just spread out to fill the space that is available to them.

The bottom right diagram is not symmetrical: in the bottom wire the electrons are at rest and in the upper wire the electrons move faster than the ions. The bottom rest diagram is obtained by considering Lorentz contraction between the two lower diagrams, as indicated by the yellow arrows. If you accept the bottom left diagram is correct, then the bottom right diagram must be correct too. Note that I could have drawn another diagram showing the frame in which the electrons in the upper wire were at rest. This diagram would look like the bottom-right diagram drawn upside down, with two static electrons in the upper wire and 14 moving rapidly to the right in the lower wire.
Per Oni said:
Referring to the bottom left picture, Biot-Savart tells me there’s a magnetic field present. Can you show me how length contraction is responsible for this magnetic field?
I'm not sure what you mean by length contraction "being responsible". We have moving electrons, i.e. a current, and therefore a magnetic field, as you say by Biot-Savart. I'm not sure what else there is to explain?
 
  • #77
DrGreg said:
I'm not sure what you mean by length contraction "being responsible".
For now I’m going to skip your first answer and look at your second.

From post #2 in this thread:
Your scenarios are explicitly covered in the section "Magnetism as a Consequence of Length Contraction".
Are they not the same issues? Are you not going to explain how magnetism is a result of length contraction?
 
  • #78
Per Oni said:
Are they not the same issues? Are you not going to explain how magnetism is a result of length contraction?
OK, I see what you are asking now -- I haven't been following this thread from the beginning.

In the bottom right diagram, a static (relative to the frame) electron near to but outside the lower wire will be attracted to it due to the net positive charge on the wire. As the electron is static, magnetism is irrelevant to it.

Translating that to the bottom left picture, the electron is now moving but the wire is not charged, so there is no electrostatic force. Nevertheless, there is still an attractive force, as we proved using the bottom right picture. The explanation for this force is magnetism. If you already knew about electrostatics and relativity but knew nothing about electromagnetism, this argument would effectively define for you what electromagnetism was.
 
  • #79
Per Oni said:
[..] From post #2 in this thread [..] Are they not the same issues? Are you not going to explain how magnetism is a result of length contraction?
The sub title on the web page that you refer to can be a bit misleading, as I also illustrated in post #33. Magnetism is not caused by length contraction.
 
  • #80
harrylin said:
There was a discussion here about almost the same topic a long time ago:
https://www.physicsforums.com/archive/index.php/t-327854.html

I agree with what I read there on the first page; I haven't read the whole discussion.
Note: Also dalespam participated. Dalespam, do you agree with your comments of then?
Yes, I went back and reviewed the thread, my comments are still correct AFAIK.
 
  • #81
Per Oni said:
Can you show me how length contraction is responsible for this magnetic field?
Careful here. You can always use relativity to explain a magnetic FORCE as a relativistic transformation of an electrostatic force in another frame, but you cannot always use relativity to explain a magnetic FIELD as a relativistic transformation of an electric field in another frame.
 
  • #82
DaleSpam said:
Careful here. You can always use relativity to explain a magnetic FORCE as a relativistic transformation of an electrostatic force in another frame, but you cannot always use relativity to explain a magnetic FIELD as a relativistic transformation of an electric field in another frame.

Shouldn't the lines of force correlate with the cause of the "FORCE" in question? That you can "always use relativity to explain a magnetic FORCE as a relativistic transformation of an electrostatic force" should imply that what is caused by magnetic flux density in one frame is cause by electric flux density in another frame. How is that the former is true but the latter not?
 
  • #83
harrylin said:
Magnetism is not caused by length contraction.
OK, I agree with this statement. We’re getting closer. What then is the cause of magnetism?

@DrGreg. It is perhaps a useful exercise to look at what happens in different frames with test charges and so on but as you stated there’s a magnetic field present in the left bottom picture. Now we have to find out why this magnetic field is there. I do not need any test charges traveling or not. Fact is we have a magnetic field. So we have to find out why the power supply had to inject an extra amount of energy. We have to find out why the energy contribution of the 2 parts of wires is increased as we increase the distance between those parts.
 
  • #84
kmarinas86 said:
That you can "always use relativity to explain a magnetic FORCE as a relativistic transformation of an electrostatic force" should imply that what is caused by magnetic flux density in one frame is cause by electric flux density in another frame. How is that the former is true but the latter not?
Remember that the magnetic field by itself does not describe the force on a charge, but you also need the velocity of the charge. The magnetic field is not proportional to the force on a charge, and knowing information about one does not uniquely determine the other without some additional information.

Furthermore, the magnetic field does not have a rest frame, whereas the magnetic force always acts on a particle which does have a rest frame. So, in general, you can always transform to a frame where the particle is at rest and be guaranteed that the magnetic force is 0, but in that frame the magnetic field may be non-zero. In general, there is not necessarily any frame where the magnetic field is 0.
 
  • #85
Per Oni said:
OK, I agree with this statement. We’re getting closer. What then is the cause of magnetism?
According to Ampere-Maxwell, magnetism is caused by the motion of the charges - perhaps we might say, by moving electric fields.

It's a bit similar to time dilation and length contraction which according to SR are caused by speed, and while for special cases all relevant speeds can be transformed away, in general this is not possible.
This it should perhaps not surprise that the same type of "absolute" vs. "relative" discussions can arise about magnetic fields as with for example the twin paradox.
 
  • #86
DaleSpam said:
Remember that the magnetic field by itself does not describe the force on a charge, but you also need the velocity of the charge. The magnetic field is not proportional to the force on a charge, and knowing information about one does not uniquely determine the other without some additional information.

Furthermore, the magnetic field does not have a rest frame, whereas the magnetic force always acts on a particle which does have a rest frame. So, in general, you can always transform to a frame where the particle is at rest and be guaranteed that the magnetic force is 0, but in that frame the magnetic field may be non-zero. In general, there is not necessarily any frame where the magnetic field is 0.

This addresses it. Thanks for answering.
 
  • #87
kmarinas86 said:
This addresses it. Thanks for answering.
I am glad, you are welcome!
 
  • #88
harrylin said:
According to Ampere-Maxwell, magnetism is caused by the motion of the charges - perhaps we might say, by moving electric fields.
That’s the way I see it.

To expand on this:
What happens when we Lorentz boost an electric field (E0)? Well we get E’=γE0. Same question regarding a magnetic field (B0). Similar, B’=γB0. How then do we get from one to the other? Clearly not by Lorentz boosting!

For the purpose of transferring between electric and magnetic fields we have 2 equations which deal with moving fields.
The following 2 formulas are copied from “Introduction to electrodynamics 3rd edition D.J.Griffiths” Formula 12.108.
Ey’=γ(Ey – vBz) and By’=γ(By + v/C^2 Ez) where v is in the x direction.
(Some time ago I lost LaTex for Microsoft Word due to a virus, does anyone know where to buy a copy?)

When we say “moving fields” I think the correct expression is “time varying fields”. But I also visualise them as moving. I’m fairly confident that these 2 formulas can also be derived from Dale’s four-vectors equation (# 30) but not sure.
 
  • #89
Per Oni said:
That’s the way I see it.

To expand on this:
What happens when we Lorentz boost an electric field (E0)? Well we get E’=γE0. Same question regarding a magnetic field (B0). Similar, B’=γB0. How then do we get from one to the other? Clearly not by Lorentz boosting!

For the purpose of transferring between electric and magnetic fields we have 2 equations which deal with moving fields.
The following 2 formulas are copied from “Introduction to electrodynamics 3rd edition D.J.Griffiths” Formula 12.108.
Ey’=γ(Ey – vBz) and By’=γ(By + v/C^2 Ez) where v is in the x direction.
(Some time ago I lost LaTex for Microsoft Word due to a virus, does anyone know where to buy a copy?)

When we say “moving fields” I think the correct expression is “time varying fields”. But I also visualise them as moving. I’m fairly confident that these 2 formulas can also be derived from Dale’s four-vectors equation (# 30) but not sure.
See post #57 and apply[tex]
\tilde{F}^{\tilde{\mu}\tilde{\nu}}=\Lambda^\tilde{\mu}_\alpha F^{\alpha\beta} \Lambda_\beta^\tilde{\nu}
[/tex]where [itex]\Lambda^\tilde{\mu}_\alpha[/itex] is the Lorentz boost matrix.
 
  • #90
harrylin said:
You refer perhaps to explanations (often accompanied by nice looking calculations) according to which magnetism is claimed to be a kind of illusion due to length contraction.

The most basic and simple case (although very high tech) that I can imagine, as it completely avoids issues with electron source and drain, is that of a closed loop superconductor in which a current is induced.

We thus start with, I think, an insulated wire containing a number of electrons N and an equal number of protons N.

I think that the following situation sketch is correct:

In the wire's rest frame:
- length contraction can play no role at all
- a magnetic field is observed

In any inertial moving frame:
- length contraction plays a role in predicting non-zero electric fields
- a magnetic field is observed that can't be transformed away

Is that correct?
Such a magnetic field looks reasonably "absolute" to me.

Harald
Thanks harrylin and DrGreg you’re a great help.
Such a magnetic field looks reasonably "absolute" to me.
Is this field absolute because we also have none moving +ve charges in the wire’s rest frame? Would it still be absolute if those charges were not present?
 
  • #91
Per Oni said:
Thanks harrylin and DrGreg you’re a great help.

Is this field absolute because we also have none moving +ve charges in the wire’s rest frame? Would it still be absolute if those charges were not present?

I don't think that the positive charges are important for the discussion. The magnetic field is here "absolute" in the sense that the magnetic field of a current loop can't be transformed away in SR. This is simply because it's impossible to transform all the velocities away in SR. Similarly, length contraction and time dilation can't be transformed away completely in such situations (see Ehrenfest paradox and twin paradox).
 
Last edited:
  • #92
Per Oni said:
Is this field absolute because we also have none moving +ve charges in the wire’s rest frame? Would it still be absolute if those charges were not present?
harrylin said:
I don't think that the positive charges are important for the discussion. The magnetic field is here "absolute" in the sense that the magnetic field of a current loop can't be transformed away in SR.
The word "absolute" doesn't merely mean that it can't be transformed away. By that definition time and length would also be absolute.

The magnetic field is relative to a given reference frame, not absolute. Just like time and length and energy and momentum and velocity and all of the other relative quantities we are familiar with.
 
Last edited:
  • #93
DaleSpam said:
The word "absolute" doesn't merely mean that it can't be transformed away. By that definition time and length would also be absolute. [..]
You misunderstood: the magnetic field is an effect that is not compared with length or time, but with length contraction and time dilation. Such effects can be transformed away in special cases.
 
  • #94
I was primarily objecting to the use of the term "absolute". Whether you are comparing it to length or to length contraction, the magnetic field is not absolute. It makes no sense to use that word to describe it.
 
  • #95
harrylin said:
This it should perhaps not surprise that the same type of "absolute" vs. "relative" discussions can arise about magnetic fields as with for example the twin paradox.

This is very nice analysis, how can time (from twin paradox) be relative when effects are totally absolute !
 
  • #96
DaleSpam said:
As I stated above, I mis-read your OP. The scenarios that I described using the LT correspond to your scenario (1) and to the LT of (1). They do not correspond to your scenario (2). I identified the modification that you would need to make to (2) in order to make it physically equivalent to (1).

They are different.

It doesn't. The link I provided uses the LT to analyze the same scenario from two different reference frames. The scenario analyzed in the link is not the same as your (1) or (2).

But if you apply charge-symmetry, then I think we should be able to transform the two scenarios.

That is, scenario(1) is exactly in conjugation with the scenario(2) according to C-symmetry.

Do you still believe the two scenarios are different and we cannot transform one to other.
 
  • #97
universal_101 said:
But if you apply charge-symmetry, then I think we should be able to transform the two scenarios.

That is, scenario(1) is exactly in conjugation with the scenario(2) according to C-symmetry.

Do you still believe the two scenarios are different and we cannot transform one to other.
Sure, but that is not a Lorentz transform. The Lorentz transform preserves charge.

Also, you would have to have completely symmetric charge carriers, i.e. no high-mass fixed "lattice" and no low-mass "free current" charge carriers. For real wires and currents you cannot transform (1) into (2) even including both charge conjugation and a boost.

EDIT: Actually, even with symmetric charge carriers you cannot change (1) into (2) because in one the test charge is at rest wrt the same polarity charge carriers and in the other the test charge is at rest wrt the opposite polarity charge carriers. Changing charge conjugation doesn't change that discrepancy.

While (1) and (2) are both perfectly valid scenarios, you cannot simply Lorentz transform from one to the other. So relativity is not going to explain them.
 
Last edited:
  • #98
harrylin said:
This is simply because it's impossible to transform all the velocities away in SR.

I go along with that.
But that would still be true even when your traveling particles are not charged. Therefore the properties you attached to the magnetic field really should be attached to a different property of physics.

Therefore, and for other reasons as well, I go along with Dale’s point of view in that magnetism is not absolute but is totally and completely dependent on the frame of reference we wish to chose. (Hoping he is happy with the way put it).
 
  • #99
Per Oni said:
Therefore, and for other reasons as well, I go along with Dale’s point of view in that magnetism is not absolute but is totally and completely dependent on the frame of reference we wish to chose. (Hoping he is happy with the way put it).
I am happy with that. I think "magnetism" refers both to the "magnetic force" and the "magnetic field", and your comment applies to both.
 
  • #100
I haven't understood and followed the entire thread and I'm sorry if this has already been answered, but if two parallel wires have current going in the same direction and then from the electrons frame the wires are positively charged and they feel a electrostatic force from the other wire, why don't the electrons feel the electrostatic force from their own wire, equalizing the electron and proton ratio?
 
  • #101
chingel said:
I haven't understood and followed the entire thread and I'm sorry if this has already been answered, but if two parallel wires have current going in the same direction and then from the electrons frame the wires are positively charged and they feel a electrostatic force from the other wire, why don't the electrons feel the electrostatic force from their own wire
They do, that is the basis of the Hall effect as described in the electron's frame. The electrostatic force from the other wire is balanced by the electrostatic force within their own wire.

chingel said:
equalizing the electron and proton ratio?
It is pointing in the wrong direction to do that.
 
  • #102
DaleSpam said:
They do, that is the basis of the Hall effect as described in the electron's frame. The electrostatic force from the other wire is balanced by the electrostatic force within their own wire.

How does the other wire's electrostatic force do that? What I mean is that in the electrons point of view, there are more protons between electrons in it's own wire than if it were electrically neutral, so why don't the protons in it's own wire pull the electrons until the charge of the wire is neutral?
 
  • #103
Does anyone know why doesn't the electrostatic force caused by the increased density of protons in its own wire in the electron's frame pull the electrons closer until the electron/proton ratio is equal in the electron's frame in its own wire? I read in this or some other thread that the experiment is set up such that the wire is neutral in the lab frame and thus the electrons cannot be pulled together, but the electrons don't know that, there must be a force on them or an explanation to make them act as such.
 
  • #104
chingel said:
Does anyone know why doesn't the electrostatic force caused by the increased density of protons in its own wire in the electron's frame pull the electrons closer until the electron/proton ratio is equal in the electron's frame in its own wire?
I am not 100% sure what you are asking, but you seem to be ascribing to the electrostatic force something that it cannot do. The electrostatic force can only move charges around, it cannot make charges appear or disappear. If the wire is charged then it is charged and there is no amount of electrostatic force that can make it otherwise.

Also, the charges are not static, so you need to think in terms of electrodynamics, not electrostatics. Fundamentally it is Maxwell's equations and the Lorentz force law that must be satisfied, not Coulomb's law except as an approximation to Maxwell and Lorentz.
 
  • #105
Has the case of a single charge been looked at yet? For example, a solitary electron is moving past an (uncharged) piece of iron and so causes a current in it due to the electron's magnetic field. How would this interaction be described from the point of view of the electron?
 
<h2>1. What is the relationship between magnetism and electrostatics?</h2><p>Magnetism is a relativistic effect of electrostatics, meaning that it is a result of the motion of charged particles. When charged particles move, they create a magnetic field, which is the force that attracts or repels other charged particles. This is known as the electromagnetic force.</p><h2>2. Why is magnetism considered an absolute phenomenon?</h2><p>Magnetism is considered an absolute phenomenon because it follows the laws of relativity and is consistent across all frames of reference. This means that the strength and direction of a magnetic field will be the same for all observers, regardless of their relative motion.</p><h2>3. How does magnetism differ from other relativistic effects?</h2><p>Unlike other relativistic effects, such as time dilation and length contraction, magnetism is not affected by the speed of the charged particles. This is because the magnetic force depends on the relative motion of the particles, not their absolute speed.</p><h2>4. Can magnetism exist without electrostatics?</h2><p>No, magnetism cannot exist without electrostatics. As mentioned before, magnetism is a result of the motion of charged particles, which is a fundamental property of electrostatics. Without charged particles, there would be no magnetic fields.</p><h2>5. How does magnetism play a role in everyday life?</h2><p>Magnetism plays a crucial role in many aspects of everyday life. It is used in technology, such as electric motors and generators, as well as in medical imaging, such as MRI machines. It also plays a role in the Earth's magnetic field, which protects us from harmful solar radiation.</p>

1. What is the relationship between magnetism and electrostatics?

Magnetism is a relativistic effect of electrostatics, meaning that it is a result of the motion of charged particles. When charged particles move, they create a magnetic field, which is the force that attracts or repels other charged particles. This is known as the electromagnetic force.

2. Why is magnetism considered an absolute phenomenon?

Magnetism is considered an absolute phenomenon because it follows the laws of relativity and is consistent across all frames of reference. This means that the strength and direction of a magnetic field will be the same for all observers, regardless of their relative motion.

3. How does magnetism differ from other relativistic effects?

Unlike other relativistic effects, such as time dilation and length contraction, magnetism is not affected by the speed of the charged particles. This is because the magnetic force depends on the relative motion of the particles, not their absolute speed.

4. Can magnetism exist without electrostatics?

No, magnetism cannot exist without electrostatics. As mentioned before, magnetism is a result of the motion of charged particles, which is a fundamental property of electrostatics. Without charged particles, there would be no magnetic fields.

5. How does magnetism play a role in everyday life?

Magnetism plays a crucial role in many aspects of everyday life. It is used in technology, such as electric motors and generators, as well as in medical imaging, such as MRI machines. It also plays a role in the Earth's magnetic field, which protects us from harmful solar radiation.

Similar threads

  • Special and General Relativity
Replies
20
Views
940
  • Special and General Relativity
Replies
34
Views
3K
  • Special and General Relativity
Replies
27
Views
2K
  • Special and General Relativity
Replies
24
Views
3K
  • Electromagnetism
Replies
17
Views
1K
  • Special and General Relativity
Replies
5
Views
1K
  • Special and General Relativity
Replies
28
Views
3K
  • Special and General Relativity
Replies
14
Views
1K
  • Special and General Relativity
Replies
1
Views
999
  • Special and General Relativity
Replies
3
Views
1K
Back
Top